Evaluating the improper integral $int_0^infty frac{xcos x-sin x}{x^3} cos(frac{x}{2}) mathrm dx $Derivative...

label a part of commutative diagram

What (if any) is the reason to buy in small local stores?

Homology of the fiber

Imaginary part of expression too difficult to calculate

How old is Nick Fury?

Unfrosted light bulb

Does the Shadow Magic sorcerer's Eyes of the Dark feature work on all Darkness spells or just his/her own?

Exit shell with shortcut (not typing exit) that closes session properly

Does fire aspect on a sword, destroy mob drops?

What is it called when someone votes for an option that's not their first choice?

Should I be concerned about student access to a test bank?

How are passwords stolen from companies if they only store hashes?

Is this Pascal's Matrix?

Hot air balloons as primitive bombers

Could any one tell what PN is this Chip? Thanks~

What kind of footwear is suitable for walking in micro gravity environment?

Did Nintendo change its mind about 68000 SNES?

What favor did Moody owe Dumbledore?

Pre-Employment Background Check With Consent For Future Checks

Should a narrator ever describe things based on a characters view instead of fact?

Why doesn't the fusion process of the sun speed up?

is this saw blade faulty?

Was World War I a war of liberals against authoritarians?

Do I need to convey a moral for each of my blog post?



Evaluating the improper integral $int_0^infty frac{xcos x-sin x}{x^3} cos(frac{x}{2}) mathrm dx $


Derivative of a Delta functionProof that $int_0^{2pi}sin nx,dx=int_0^{2pi}cos nx,dx=0$Hints on evaluating this complex integral?How to simplify the integral of $intfrac{cos(8x)}{cos(4x)+sin(4x)}dx$?About the integral $int_{0}^{1}frac{log(x)log^2(1+x)}{x},dx$Definite integral $int_{-1}^{1} e^{frac{1}{x^2-1}}cos{ax},dx$Trigonometric Definite IntegralEvaluate the integral $int_0^infty frac{sin(ax)}{x} frac{sin(bx)}{x}e^{-kx}dx$Integral $int_0^infty frac{x^{2j}mathrm dx}{(x^4+2ax^2+1)^{n+1}}$Solve $intlimits_{0}^{infty}frac{sin(x)}{xe^x} dx.$













15












$begingroup$


I've been working through the following integral and am stumped:



$$int_0^infty frac{xcos x-sin x}{x^3}cosleft(frac{x}{2}right)mathrm dx$$



Given the questions in my class that have proceeded and followed this integral, I believe that this is some form of Fourier transform/integral. However, it doesn't look like any of the content surrounding it. That is, there is no $e^{-ikx}$ or $g(k)$ or anything else that I'm familiar with.



I know that it's an even function, but that's about as far as I can get. If I try to split it over the subtraction, I get two non-converging integrals, so that wasn't much help either. I've been throwing lots of trig identities at it but nothing familiar has appeared yet.



Any help would be greatly appreciated.
Thank you.










share|cite|improve this question











$endgroup$





This question had a bounty worth +150
reputation from Tyler6 that ended 1 hour ago. Grace period ends in 22 hours


The current answer(s) are out-of-date and require revision given recent changes.


Would like an answer that used Fourier Transforms as initially requested
















  • $begingroup$
    @Tyler6 : You can use $displaystyleintfrac{xcos x - sin x}{x^3}cosfrac{x}{2}dx = -frac{3}{16}left( intlimits_0^{x/2}frac{sin x}{x} + intlimits_0^{3x/2}frac{sin x}{x} right) -frac{cos^2frac{x}{2}}{x^2}left(frac{x}{2}cosfrac{x}{2} - sinfrac{x}{2}right) + C$ $~~$ Why Fourier Transforms ?
    $endgroup$
    – user90369
    Mar 13 at 11:46


















15












$begingroup$


I've been working through the following integral and am stumped:



$$int_0^infty frac{xcos x-sin x}{x^3}cosleft(frac{x}{2}right)mathrm dx$$



Given the questions in my class that have proceeded and followed this integral, I believe that this is some form of Fourier transform/integral. However, it doesn't look like any of the content surrounding it. That is, there is no $e^{-ikx}$ or $g(k)$ or anything else that I'm familiar with.



I know that it's an even function, but that's about as far as I can get. If I try to split it over the subtraction, I get two non-converging integrals, so that wasn't much help either. I've been throwing lots of trig identities at it but nothing familiar has appeared yet.



Any help would be greatly appreciated.
Thank you.










share|cite|improve this question











$endgroup$





This question had a bounty worth +150
reputation from Tyler6 that ended 1 hour ago. Grace period ends in 22 hours


The current answer(s) are out-of-date and require revision given recent changes.


Would like an answer that used Fourier Transforms as initially requested
















  • $begingroup$
    @Tyler6 : You can use $displaystyleintfrac{xcos x - sin x}{x^3}cosfrac{x}{2}dx = -frac{3}{16}left( intlimits_0^{x/2}frac{sin x}{x} + intlimits_0^{3x/2}frac{sin x}{x} right) -frac{cos^2frac{x}{2}}{x^2}left(frac{x}{2}cosfrac{x}{2} - sinfrac{x}{2}right) + C$ $~~$ Why Fourier Transforms ?
    $endgroup$
    – user90369
    Mar 13 at 11:46
















15












15








15


4



$begingroup$


I've been working through the following integral and am stumped:



$$int_0^infty frac{xcos x-sin x}{x^3}cosleft(frac{x}{2}right)mathrm dx$$



Given the questions in my class that have proceeded and followed this integral, I believe that this is some form of Fourier transform/integral. However, it doesn't look like any of the content surrounding it. That is, there is no $e^{-ikx}$ or $g(k)$ or anything else that I'm familiar with.



I know that it's an even function, but that's about as far as I can get. If I try to split it over the subtraction, I get two non-converging integrals, so that wasn't much help either. I've been throwing lots of trig identities at it but nothing familiar has appeared yet.



Any help would be greatly appreciated.
Thank you.










share|cite|improve this question











$endgroup$




I've been working through the following integral and am stumped:



$$int_0^infty frac{xcos x-sin x}{x^3}cosleft(frac{x}{2}right)mathrm dx$$



Given the questions in my class that have proceeded and followed this integral, I believe that this is some form of Fourier transform/integral. However, it doesn't look like any of the content surrounding it. That is, there is no $e^{-ikx}$ or $g(k)$ or anything else that I'm familiar with.



I know that it's an even function, but that's about as far as I can get. If I try to split it over the subtraction, I get two non-converging integrals, so that wasn't much help either. I've been throwing lots of trig identities at it but nothing familiar has appeared yet.



Any help would be greatly appreciated.
Thank you.







calculus integration fourier-analysis improper-integrals fourier-transform






share|cite|improve this question















share|cite|improve this question













share|cite|improve this question




share|cite|improve this question








edited Mar 13 at 18:47









mrtaurho

6,00551641




6,00551641










asked Oct 29 '16 at 5:07









Doe aDoe a

996




996






This question had a bounty worth +150
reputation from Tyler6 that ended 1 hour ago. Grace period ends in 22 hours


The current answer(s) are out-of-date and require revision given recent changes.


Would like an answer that used Fourier Transforms as initially requested








This question had a bounty worth +150
reputation from Tyler6 that ended 1 hour ago. Grace period ends in 22 hours


The current answer(s) are out-of-date and require revision given recent changes.


Would like an answer that used Fourier Transforms as initially requested














  • $begingroup$
    @Tyler6 : You can use $displaystyleintfrac{xcos x - sin x}{x^3}cosfrac{x}{2}dx = -frac{3}{16}left( intlimits_0^{x/2}frac{sin x}{x} + intlimits_0^{3x/2}frac{sin x}{x} right) -frac{cos^2frac{x}{2}}{x^2}left(frac{x}{2}cosfrac{x}{2} - sinfrac{x}{2}right) + C$ $~~$ Why Fourier Transforms ?
    $endgroup$
    – user90369
    Mar 13 at 11:46




















  • $begingroup$
    @Tyler6 : You can use $displaystyleintfrac{xcos x - sin x}{x^3}cosfrac{x}{2}dx = -frac{3}{16}left( intlimits_0^{x/2}frac{sin x}{x} + intlimits_0^{3x/2}frac{sin x}{x} right) -frac{cos^2frac{x}{2}}{x^2}left(frac{x}{2}cosfrac{x}{2} - sinfrac{x}{2}right) + C$ $~~$ Why Fourier Transforms ?
    $endgroup$
    – user90369
    Mar 13 at 11:46


















$begingroup$
@Tyler6 : You can use $displaystyleintfrac{xcos x - sin x}{x^3}cosfrac{x}{2}dx = -frac{3}{16}left( intlimits_0^{x/2}frac{sin x}{x} + intlimits_0^{3x/2}frac{sin x}{x} right) -frac{cos^2frac{x}{2}}{x^2}left(frac{x}{2}cosfrac{x}{2} - sinfrac{x}{2}right) + C$ $~~$ Why Fourier Transforms ?
$endgroup$
– user90369
Mar 13 at 11:46






$begingroup$
@Tyler6 : You can use $displaystyleintfrac{xcos x - sin x}{x^3}cosfrac{x}{2}dx = -frac{3}{16}left( intlimits_0^{x/2}frac{sin x}{x} + intlimits_0^{3x/2}frac{sin x}{x} right) -frac{cos^2frac{x}{2}}{x^2}left(frac{x}{2}cosfrac{x}{2} - sinfrac{x}{2}right) + C$ $~~$ Why Fourier Transforms ?
$endgroup$
– user90369
Mar 13 at 11:46












6 Answers
6






active

oldest

votes


















9












$begingroup$

$newcommand{bbx}[1]{,bbox[8px,border:1px groove navy]{{#1}},}
newcommand{braces}[1]{leftlbrace,{#1},rightrbrace}
newcommand{bracks}[1]{leftlbrack,{#1},rightrbrack}
newcommand{dd}{mathrm{d}}
newcommand{ds}[1]{displaystyle{#1}}
newcommand{expo}[1]{,mathrm{e}^{#1},}
newcommand{ic}{mathrm{i}}
newcommand{mc}[1]{mathcal{#1}}
newcommand{mrm}[1]{mathrm{#1}}
newcommand{pars}[1]{left(,{#1},right)}
newcommand{partiald}[3][]{frac{partial^{#1} #2}{partial #3^{#1}}}
newcommand{root}[2][]{,sqrt[#1]{,{#2},},}
newcommand{totald}[3][]{frac{mathrm{d}^{#1} #2}{mathrm{d} #3^{#1}}}
newcommand{verts}[1]{leftvert,{#1},rightvert}$
begin{align}
&int_{0}^{infty}{xcospars{x} - sinpars{x} over x^{3}},
cospars{x over 2},dd x =
int_{0}^{infty}{xbracks{1 - 2sin^{2}pars{x/2}} - sinpars{x} over x^{3}},cospars{x over 2},dd x
\[5mm] & =
{1 over 2}int_{0}^{infty}{2xcospars{x/2} - 2xsinpars{x}sinpars{x/2} -
2sinpars{x}cospars{x/2} over x^{3}},,,dd x
\[5mm] & =
{1 over 2}int_{0}^{infty}{xcospars{x/2} + xcospars{3x/2} -
sinpars{3x/2} - sinpars{x/2} over x^{3}},,,dd x
\[1cm] & =
-,{1 over 2}int_{0}^{infty}{1 - cospars{x/2} over x^{2}},dd x -
{1 over 2}int_{0}^{infty}{1 - cospars{3x/2} over x^{2}},dd x
\[5mm] & -
{1 over 4}int_{x = 0}^{x to infty}bracks{2x - sinpars{3x/2} - sinpars{x/2}},ddpars{1 over x^{2}}
end{align}




Integrating by parts the last integral:
begin{align}
&int_{0}^{infty}{xcospars{x} - sinpars{x} over x^{3}},
cospars{x over 2},dd x =
\[5mm] & =
-,{1 over 2}int_{0}^{infty}{1 - cospars{x/2} over x^{2}},dd x -
{1 over 2}int_{0}^{infty}{1 - cospars{3x/2} over x^{2}},dd x
\[5mm] & +
{1 over 4}int_{x = 0}^{infty}{2 - 3cospars{3x/2}/2 - cospars{x/2}/2 over x^{2}},dd x
\[1cm] & =
-,{1 over 2}int_{0}^{infty}{1 - cospars{x/2} over x^{2}},dd x -
{1 over 2}int_{0}^{infty}{1 - cospars{3x/2} over x^{2}},dd x
\[5mm] & +
{3 over 8}int_{0}^{infty}{1 - cospars{3x/2} over x^{2}},dd x +
{1 over 8}int_{0}^{infty}{1 - cospars{x/2} over x^{2}},dd x
\[1cm] & =
-,{3 over 8}int_{0}^{infty}{1 - cospars{x/2} over x^{2}},dd x
-,{1 over 8}int_{0}^{infty}{1 - cospars{3x/2} over x^{2}},dd x
\[5mm] & =
-,{3 over 16}int_{0}^{infty}{1 - cospars{x} over x^{2}},dd x
-,{3 over 16}int_{0}^{infty}{1 - cospars{x} over x^{2}},dd x =
-,{3 over 8}
int_{0}^{infty}{1 - cospars{x} over x^{2}},dd x
\[5mm] & =
-,{3 over 4}int_{0}^{infty}{sin^{2}pars{x/2} over x^{2}},dd x =
-,{3 over 8}
underbrace{int_{0}^{infty}{sin^{2}pars{x} over x^{2}},dd x}
_{ds{= {pi over 2}}} =
bbox[#ffe,10px,border:1px dotted navy]{ds{-,{3 over 16},pi}}
end{align}


By integrating by parts:
$ds{int_{0}^{infty}{sin^{2}pars{x} over x^{2}},dd x =
int_{0}^{infty}{sinpars{x} over x},dd x = {1 over 2},pi}$.







share|cite|improve this answer









$endgroup$





















    7












    $begingroup$

    $$
    begin{align}
    &int_0^inftyfrac{xcos(x)-sin(x)}{x^3}cosleft(frac{x}{2}right),mathrm{d}xtag{1}\
    &=int_0^inftyfrac{xleft(cosleft(frac32xright)+cosleft(frac12xright)right)-left(sinleft(frac32xright)+sinleft(frac12xright)right)}{2x^3},mathrm{d}xtag{2}\
    &=-int_0^inftyfrac{xleft(cosleft(tfrac32xright)+cosleft(tfrac12xright)right)-left(sinleft(tfrac32xright)+sinleft(tfrac12xright)right)}{4},mathrm{d}x^{-2}tag{3}\
    &=int_0^inftyfrac{left(frac12cosleft(frac12xright)-frac12cosleft(frac32xright)right)-xleft(frac32sinleft(frac32xright)+frac12sinleft(frac12xright)right)}{4x^2},mathrm{d}xtag{4}\
    &=int_0^inftyleft(frac{1-cosleft(frac32xright)}{8x^2}-frac{1-cosleft(frac12xright)}{8x^2}-frac{3sinleft(frac32xright)}{8x}-frac{sinleft(frac12xright)}{8x}right)mathrm{d}x
    tag{5}\
    &=int_0^inftyleft(frac{3(1-cos(x))}{16x^2}-frac{1-cos(x)}{16x^2}-frac{3sin(x)}{8x}-frac{sin(x)}{8x}right)mathrm{d}x
    tag{6}\
    &=int_0^inftyleft(frac{3sin(x)}{16x}-frac{sin(x)}{16x}-frac{3sin(x)}{8x}-frac{sin(x)}{8x}right)mathrm{d}x
    tag{7}\
    &=-frac38int_0^inftyfrac{sin(x)}{x},mathrm{d}xtag{8}\
    &=-frac{3pi}{16}tag{9}
    end{align}
    $$
    Explanation:

    $(2)$: trigonometric product formulas

    $(3)$: prepare to integrate by parts

    $(4)$: integrate by parts

    $(5)$: separate integrals

    $(6)$: substitute $xmapsto2x$ and $xmapstofrac23x$

    $(7)$: integrate by parts

    $(8)$: combine

    $(9)$: $int_0^inftyfrac{sin(x)}{x},mathrm{d}x=fracpi2$






    share|cite|improve this answer











    $endgroup$













    • $begingroup$
      I see that this may be similar to Felix Marin's answer, but I think the path may be different enough, and possibly simpler, to warrant leaving it.
      $endgroup$
      – robjohn
      Oct 29 '16 at 13:53





















    6












    $begingroup$

    We can also use contour integration.



    $$ begin{align} &int_{0}^{infty} frac{x cos x - sin x}{x^{3}} , cos left(frac{x}{2} right) , dx \ &= frac{1}{2} int_{-infty}^{infty} frac{x left(frac{e^{ix}+e^{-ix}}{2} right) -frac{e^{ix}-e^{-ix}}{2i}}{x^{3}} left(frac{e^{ix/2}+e^{-ix/2}}{2} right) , dx \ &= frac{1}{2} lim_{epsilon to 0^{+}} int_{-infty}^{infty} frac{x left(frac{e^{ix}+e^{-ix}}{2} right) -frac{e^{ix}-e^{-ix}}{2i}}{(x- i epsilon)^{3}} left(frac{e^{ix/2}+e^{-ix/2}}{2} right) , dx \ &= frac{1}{8} lim_{epsilon to 0^{+}} int_{-infty}^{infty}frac{(x+i)(e^{3ix/2}+e^{ix/2})}{(x- iepsilon)^{3}} , dx + frac{1}{8} lim_{epsilon to 0^{+}} int_{-infty}^{infty} frac{(x-i)(e^{-ix/2}+e^{-3ix/2})}{(x- iepsilon)^{3}} , dx \ &=frac{1}{8} lim_{epsilon to 0^{+}} 2 pi i , text{Res} left[frac{(z+i)(e^{3iz/2}+e^{iz/2})}{(z- i epsilon)^{3}} , iepsilon right] + 0 tag{1} \ &= frac{1}{8} lim_{epsilon to 0^{+}} , 2 pi i , frac{1}{2!} lim_{z to i epsilon}frac{d^{2}}{dz^{2}} , (z+i)(e^{3iz/2}+e^{iz/2}) \ &= frac{1}{8} lim_{epsilon to 0^{+}} frac{pi}{4} , e^{-3 epsilon/2} left((epsilon-3) e^{epsilon} + 9 epsilon -3 right) \ &= - frac{3 pi}{16} end{align}$$





    $(1)$ The second integral vanishes since the function $ displaystyle frac{(z-i)(e^{-iz/2}+e^{-3iz/2})}{(z- iepsilon)^{3}} $ is analytic in the lower half-plane where $left| e^{iaz} right| le 1$ if $a le 0$.






    share|cite|improve this answer











    $endgroup$





















      4












      $begingroup$

      Inspired by Felix Marin's calculation using integration by parts.



      Observe
      begin{align}
      int^infty_0 frac{xcos x-sin x}{x^3}cosfrac{x}{2} dx=& frac{1}{2}int^infty_{-infty} frac{xcos x-sin x}{x^3}e^{ix/2} dx\
      =& frac{-1}{4} int^infty_{-infty} [xcos x-sin x] e^{ix/2} dleft(frac{1}{x^2} right).
      end{align}
      Using integration by parts, we have
      begin{align}
      frac{-1}{4} int^infty_{-infty} [xcos x-sin x] e^{ix/2} dleft(frac{1}{x^2} right)=& frac{1}{4} int^infty_{-infty}d([xcos x-sin x]e^{ix/2}) frac{1}{x^2}\
      =& frac{-1}{4} int^infty_{-infty}frac{sin x}{x}e^{ix/2} dx + frac{i}{8} int^infty_{-infty} frac{xcos x-sin x}{x^2}e^{ix/2} dx.
      end{align}
      Now, observe
      begin{align}
      int^infty_{-infty}frac{sin x}{x}e^{ix/2} dx = mathcal{F}^{-1}left[operatorname{sincleft(frac{x}{pi}right)}right]left(frac{1}{2}right) = pi mathcal{F}^{-1}[operatorname{sinc}]left( frac{1}{2pi}right) = pi.
      end{align}



      Next, observe
      begin{align}
      int^infty_{-infty} frac{xcos x-sin x}{x^2} e^{ix/2} dx =& int^infty_{-infty} frac{d}{dx}left( frac{sin x}{x}right) e^{ix/2} dx\
      =& -frac{i}{2}int^infty_{-infty}frac{sin x}{x} e^{ix/2} dx = -frac{ipi}{2}.
      end{align}



      Hence combining everything yields
      begin{align}
      int^infty_0 frac{xcos x-sin x}{x^3} cos frac{x}{2} dx = -frac{pi}{4} + frac{pi}{16} = -frac{3pi}{16}.
      end{align}






      share|cite|improve this answer











      $endgroup$





















        3












        $begingroup$

        Integration by parts can be performed for the indefinite integral, using relations




        $$dfrac{xcos x-sin x}{x^2} = left(dfrac{sin x}{x}right)',quad
        sin^3 z =frac{3sin z-sin3z}4,quad cos^3z=frac{3cos
        z+cos3z}4.$$




        One can get
        begin{align}
        &int dfrac{xcos x-sin x}{x^3},cosdfrac x2, mathrm dx
        = intdfrac1{4sin frac x2},mathrm dleft(dfrac{sin x}{x}right)^2 \[4pt]
        &=dfrac1{4sin frac x2}left(dfrac{sin x}{x}right)^2
        +intleft(dfrac{sin x}{x}right)^2dfrac{cosfrac x2}{8sin^2 frac x2},mathrm dx
        =dfrac1{x^2}sinfrac x2,cos^2 frac x2
        +dfrac12intdfrac{cos^3frac x2}{x^2},mathrm dx\[4pt]
        &=dfrac1{x^2}sinfrac x2
        -dfrac1{4x^2}left(3sinfrac x2-sin frac {3x}2right)
        +dfrac18intdfrac{3cosfrac x2+cosfrac{3x}2}{x^2},mathrm dx\[4pt]
        &=dfrac1{4x^2}left(sinfrac x2+sin frac {3x}2right)
        -dfrac18intleft(3cosfrac x2+cosfrac{3x}2right),mathrm dfrac1x\[4pt]
        &=dfrac1{8x^2}left(2sinfrac x2+2sinfrac{3x}2-3xcosfrac {x}2-xcos frac {3x}2right)
        -dfrac3{16}intfrac1xleft(sinfrac x2+sinfrac{3x}2right),mathrm dx.
        end{align}

        Since
        $$limlimits_{xto0}dfrac{2sinfrac x2+2sinfrac{3x}2-3xcosfrac {x}2-xcosfrac {3x}2}{8x^2}
        = limlimits_{xto0}dfrac{2frac x2+2frac{3x}2-3x-x+O(x^3)}{8x^2}=0,$$




        $$intlimits_{0}^{infty} dfrac{sin x}x,mathrm dx =fracpi2,$$




        then
        $$intlimits_{0}^{infty} dfrac{xcos x-sin x}{x^3},cosdfrac x2, mathrm dx
        =-frac3{16}left(intlimits_{0}^{infty} dfrac{sinfrac x2}{frac x2},mathrm dfrac x2+intlimits_{0}^{infty} dfrac{sin frac{3x}2}{frac{3x}2},mathrm dfrac{3x}2right) = color{green}{mathbf{-frac{3pi}{16}}}.$$






        share|cite|improve this answer











        $endgroup$









        • 1




          $begingroup$
          elegant approach
          $endgroup$
          – G Cab
          Mar 15 at 22:53










        • $begingroup$
          @GCab Thanks. I tried to understand the essence of the problem.
          $endgroup$
          – Yuri Negometyanov
          Mar 15 at 23:06



















        1












        $begingroup$

        I use the Laplace transform method instead of Fourier because the first one is the main tool for an electrician (who I am) and the Laplace transform is very similar to the Fourier transform.



        By definition of the Laplace transform:



        $mathcal{L}f(x)=int_{0}^{infty}e^{-sx}f(x)dx=F(s)$



        $mathcal{L}g(x)=int_{0}^{infty}e^{-sx}g(x)dx=G(s)$



        Now we use the following equation from the Laplace transform theory:



        $$int_{0}^{infty}F(x)g(x)dx=int_{0}^{infty}G(x)f(x)dx$$



        and apply it to the required integral.



        We take



        $F(x)=frac{1}{x^3}$



        $g(x)=xcos xcos frac{x}{2}-sin xcos frac{x}{2}$



        and get after taking the Laplace and inverse Laplace transforms



        $f(x)=frac{x^2}{2}$



        $G(x)=frac{4x^2-3}{(4x^2+1)^2}-frac{4x^2+45}{(4x^2+9)^2}$



        Placing these results into the relationship above we arrive at the next expression for the required integral:



        $$I=frac{1}{2}int_{0}^{infty}x^2left [ frac{4x^2-3}{(4x^2+1)^2}-frac{4x^2+45}{(4x^2+9)^2} right ]dx$$



        $I=left [- frac{3}{16}arctan 2x- frac{3}{16}arctanfrac{2x}{3}+frac{9x}{4(4x^2+9)^2}+frac{x}{4(4x^2+1)^2}right ]_{0}^{infty}=$



        $=- frac{3}{16}frac{pi}{2}- frac{3}{16}frac{pi}{2}=- frac{3pi}{16}$






        share|cite|improve this answer









        $endgroup$













          Your Answer





          StackExchange.ifUsing("editor", function () {
          return StackExchange.using("mathjaxEditing", function () {
          StackExchange.MarkdownEditor.creationCallbacks.add(function (editor, postfix) {
          StackExchange.mathjaxEditing.prepareWmdForMathJax(editor, postfix, [["$", "$"], ["\\(","\\)"]]);
          });
          });
          }, "mathjax-editing");

          StackExchange.ready(function() {
          var channelOptions = {
          tags: "".split(" "),
          id: "69"
          };
          initTagRenderer("".split(" "), "".split(" "), channelOptions);

          StackExchange.using("externalEditor", function() {
          // Have to fire editor after snippets, if snippets enabled
          if (StackExchange.settings.snippets.snippetsEnabled) {
          StackExchange.using("snippets", function() {
          createEditor();
          });
          }
          else {
          createEditor();
          }
          });

          function createEditor() {
          StackExchange.prepareEditor({
          heartbeatType: 'answer',
          autoActivateHeartbeat: false,
          convertImagesToLinks: true,
          noModals: true,
          showLowRepImageUploadWarning: true,
          reputationToPostImages: 10,
          bindNavPrevention: true,
          postfix: "",
          imageUploader: {
          brandingHtml: "Powered by u003ca class="icon-imgur-white" href="https://imgur.com/"u003eu003c/au003e",
          contentPolicyHtml: "User contributions licensed under u003ca href="https://creativecommons.org/licenses/by-sa/3.0/"u003ecc by-sa 3.0 with attribution requiredu003c/au003e u003ca href="https://stackoverflow.com/legal/content-policy"u003e(content policy)u003c/au003e",
          allowUrls: true
          },
          noCode: true, onDemand: true,
          discardSelector: ".discard-answer"
          ,immediatelyShowMarkdownHelp:true
          });


          }
          });














          draft saved

          draft discarded


















          StackExchange.ready(
          function () {
          StackExchange.openid.initPostLogin('.new-post-login', 'https%3a%2f%2fmath.stackexchange.com%2fquestions%2f1989935%2fevaluating-the-improper-integral-int-0-infty-fracx-cos-x-sin-xx3-cos%23new-answer', 'question_page');
          }
          );

          Post as a guest















          Required, but never shown

























          6 Answers
          6






          active

          oldest

          votes








          6 Answers
          6






          active

          oldest

          votes









          active

          oldest

          votes






          active

          oldest

          votes









          9












          $begingroup$

          $newcommand{bbx}[1]{,bbox[8px,border:1px groove navy]{{#1}},}
          newcommand{braces}[1]{leftlbrace,{#1},rightrbrace}
          newcommand{bracks}[1]{leftlbrack,{#1},rightrbrack}
          newcommand{dd}{mathrm{d}}
          newcommand{ds}[1]{displaystyle{#1}}
          newcommand{expo}[1]{,mathrm{e}^{#1},}
          newcommand{ic}{mathrm{i}}
          newcommand{mc}[1]{mathcal{#1}}
          newcommand{mrm}[1]{mathrm{#1}}
          newcommand{pars}[1]{left(,{#1},right)}
          newcommand{partiald}[3][]{frac{partial^{#1} #2}{partial #3^{#1}}}
          newcommand{root}[2][]{,sqrt[#1]{,{#2},},}
          newcommand{totald}[3][]{frac{mathrm{d}^{#1} #2}{mathrm{d} #3^{#1}}}
          newcommand{verts}[1]{leftvert,{#1},rightvert}$
          begin{align}
          &int_{0}^{infty}{xcospars{x} - sinpars{x} over x^{3}},
          cospars{x over 2},dd x =
          int_{0}^{infty}{xbracks{1 - 2sin^{2}pars{x/2}} - sinpars{x} over x^{3}},cospars{x over 2},dd x
          \[5mm] & =
          {1 over 2}int_{0}^{infty}{2xcospars{x/2} - 2xsinpars{x}sinpars{x/2} -
          2sinpars{x}cospars{x/2} over x^{3}},,,dd x
          \[5mm] & =
          {1 over 2}int_{0}^{infty}{xcospars{x/2} + xcospars{3x/2} -
          sinpars{3x/2} - sinpars{x/2} over x^{3}},,,dd x
          \[1cm] & =
          -,{1 over 2}int_{0}^{infty}{1 - cospars{x/2} over x^{2}},dd x -
          {1 over 2}int_{0}^{infty}{1 - cospars{3x/2} over x^{2}},dd x
          \[5mm] & -
          {1 over 4}int_{x = 0}^{x to infty}bracks{2x - sinpars{3x/2} - sinpars{x/2}},ddpars{1 over x^{2}}
          end{align}




          Integrating by parts the last integral:
          begin{align}
          &int_{0}^{infty}{xcospars{x} - sinpars{x} over x^{3}},
          cospars{x over 2},dd x =
          \[5mm] & =
          -,{1 over 2}int_{0}^{infty}{1 - cospars{x/2} over x^{2}},dd x -
          {1 over 2}int_{0}^{infty}{1 - cospars{3x/2} over x^{2}},dd x
          \[5mm] & +
          {1 over 4}int_{x = 0}^{infty}{2 - 3cospars{3x/2}/2 - cospars{x/2}/2 over x^{2}},dd x
          \[1cm] & =
          -,{1 over 2}int_{0}^{infty}{1 - cospars{x/2} over x^{2}},dd x -
          {1 over 2}int_{0}^{infty}{1 - cospars{3x/2} over x^{2}},dd x
          \[5mm] & +
          {3 over 8}int_{0}^{infty}{1 - cospars{3x/2} over x^{2}},dd x +
          {1 over 8}int_{0}^{infty}{1 - cospars{x/2} over x^{2}},dd x
          \[1cm] & =
          -,{3 over 8}int_{0}^{infty}{1 - cospars{x/2} over x^{2}},dd x
          -,{1 over 8}int_{0}^{infty}{1 - cospars{3x/2} over x^{2}},dd x
          \[5mm] & =
          -,{3 over 16}int_{0}^{infty}{1 - cospars{x} over x^{2}},dd x
          -,{3 over 16}int_{0}^{infty}{1 - cospars{x} over x^{2}},dd x =
          -,{3 over 8}
          int_{0}^{infty}{1 - cospars{x} over x^{2}},dd x
          \[5mm] & =
          -,{3 over 4}int_{0}^{infty}{sin^{2}pars{x/2} over x^{2}},dd x =
          -,{3 over 8}
          underbrace{int_{0}^{infty}{sin^{2}pars{x} over x^{2}},dd x}
          _{ds{= {pi over 2}}} =
          bbox[#ffe,10px,border:1px dotted navy]{ds{-,{3 over 16},pi}}
          end{align}


          By integrating by parts:
          $ds{int_{0}^{infty}{sin^{2}pars{x} over x^{2}},dd x =
          int_{0}^{infty}{sinpars{x} over x},dd x = {1 over 2},pi}$.







          share|cite|improve this answer









          $endgroup$


















            9












            $begingroup$

            $newcommand{bbx}[1]{,bbox[8px,border:1px groove navy]{{#1}},}
            newcommand{braces}[1]{leftlbrace,{#1},rightrbrace}
            newcommand{bracks}[1]{leftlbrack,{#1},rightrbrack}
            newcommand{dd}{mathrm{d}}
            newcommand{ds}[1]{displaystyle{#1}}
            newcommand{expo}[1]{,mathrm{e}^{#1},}
            newcommand{ic}{mathrm{i}}
            newcommand{mc}[1]{mathcal{#1}}
            newcommand{mrm}[1]{mathrm{#1}}
            newcommand{pars}[1]{left(,{#1},right)}
            newcommand{partiald}[3][]{frac{partial^{#1} #2}{partial #3^{#1}}}
            newcommand{root}[2][]{,sqrt[#1]{,{#2},},}
            newcommand{totald}[3][]{frac{mathrm{d}^{#1} #2}{mathrm{d} #3^{#1}}}
            newcommand{verts}[1]{leftvert,{#1},rightvert}$
            begin{align}
            &int_{0}^{infty}{xcospars{x} - sinpars{x} over x^{3}},
            cospars{x over 2},dd x =
            int_{0}^{infty}{xbracks{1 - 2sin^{2}pars{x/2}} - sinpars{x} over x^{3}},cospars{x over 2},dd x
            \[5mm] & =
            {1 over 2}int_{0}^{infty}{2xcospars{x/2} - 2xsinpars{x}sinpars{x/2} -
            2sinpars{x}cospars{x/2} over x^{3}},,,dd x
            \[5mm] & =
            {1 over 2}int_{0}^{infty}{xcospars{x/2} + xcospars{3x/2} -
            sinpars{3x/2} - sinpars{x/2} over x^{3}},,,dd x
            \[1cm] & =
            -,{1 over 2}int_{0}^{infty}{1 - cospars{x/2} over x^{2}},dd x -
            {1 over 2}int_{0}^{infty}{1 - cospars{3x/2} over x^{2}},dd x
            \[5mm] & -
            {1 over 4}int_{x = 0}^{x to infty}bracks{2x - sinpars{3x/2} - sinpars{x/2}},ddpars{1 over x^{2}}
            end{align}




            Integrating by parts the last integral:
            begin{align}
            &int_{0}^{infty}{xcospars{x} - sinpars{x} over x^{3}},
            cospars{x over 2},dd x =
            \[5mm] & =
            -,{1 over 2}int_{0}^{infty}{1 - cospars{x/2} over x^{2}},dd x -
            {1 over 2}int_{0}^{infty}{1 - cospars{3x/2} over x^{2}},dd x
            \[5mm] & +
            {1 over 4}int_{x = 0}^{infty}{2 - 3cospars{3x/2}/2 - cospars{x/2}/2 over x^{2}},dd x
            \[1cm] & =
            -,{1 over 2}int_{0}^{infty}{1 - cospars{x/2} over x^{2}},dd x -
            {1 over 2}int_{0}^{infty}{1 - cospars{3x/2} over x^{2}},dd x
            \[5mm] & +
            {3 over 8}int_{0}^{infty}{1 - cospars{3x/2} over x^{2}},dd x +
            {1 over 8}int_{0}^{infty}{1 - cospars{x/2} over x^{2}},dd x
            \[1cm] & =
            -,{3 over 8}int_{0}^{infty}{1 - cospars{x/2} over x^{2}},dd x
            -,{1 over 8}int_{0}^{infty}{1 - cospars{3x/2} over x^{2}},dd x
            \[5mm] & =
            -,{3 over 16}int_{0}^{infty}{1 - cospars{x} over x^{2}},dd x
            -,{3 over 16}int_{0}^{infty}{1 - cospars{x} over x^{2}},dd x =
            -,{3 over 8}
            int_{0}^{infty}{1 - cospars{x} over x^{2}},dd x
            \[5mm] & =
            -,{3 over 4}int_{0}^{infty}{sin^{2}pars{x/2} over x^{2}},dd x =
            -,{3 over 8}
            underbrace{int_{0}^{infty}{sin^{2}pars{x} over x^{2}},dd x}
            _{ds{= {pi over 2}}} =
            bbox[#ffe,10px,border:1px dotted navy]{ds{-,{3 over 16},pi}}
            end{align}


            By integrating by parts:
            $ds{int_{0}^{infty}{sin^{2}pars{x} over x^{2}},dd x =
            int_{0}^{infty}{sinpars{x} over x},dd x = {1 over 2},pi}$.







            share|cite|improve this answer









            $endgroup$
















              9












              9








              9





              $begingroup$

              $newcommand{bbx}[1]{,bbox[8px,border:1px groove navy]{{#1}},}
              newcommand{braces}[1]{leftlbrace,{#1},rightrbrace}
              newcommand{bracks}[1]{leftlbrack,{#1},rightrbrack}
              newcommand{dd}{mathrm{d}}
              newcommand{ds}[1]{displaystyle{#1}}
              newcommand{expo}[1]{,mathrm{e}^{#1},}
              newcommand{ic}{mathrm{i}}
              newcommand{mc}[1]{mathcal{#1}}
              newcommand{mrm}[1]{mathrm{#1}}
              newcommand{pars}[1]{left(,{#1},right)}
              newcommand{partiald}[3][]{frac{partial^{#1} #2}{partial #3^{#1}}}
              newcommand{root}[2][]{,sqrt[#1]{,{#2},},}
              newcommand{totald}[3][]{frac{mathrm{d}^{#1} #2}{mathrm{d} #3^{#1}}}
              newcommand{verts}[1]{leftvert,{#1},rightvert}$
              begin{align}
              &int_{0}^{infty}{xcospars{x} - sinpars{x} over x^{3}},
              cospars{x over 2},dd x =
              int_{0}^{infty}{xbracks{1 - 2sin^{2}pars{x/2}} - sinpars{x} over x^{3}},cospars{x over 2},dd x
              \[5mm] & =
              {1 over 2}int_{0}^{infty}{2xcospars{x/2} - 2xsinpars{x}sinpars{x/2} -
              2sinpars{x}cospars{x/2} over x^{3}},,,dd x
              \[5mm] & =
              {1 over 2}int_{0}^{infty}{xcospars{x/2} + xcospars{3x/2} -
              sinpars{3x/2} - sinpars{x/2} over x^{3}},,,dd x
              \[1cm] & =
              -,{1 over 2}int_{0}^{infty}{1 - cospars{x/2} over x^{2}},dd x -
              {1 over 2}int_{0}^{infty}{1 - cospars{3x/2} over x^{2}},dd x
              \[5mm] & -
              {1 over 4}int_{x = 0}^{x to infty}bracks{2x - sinpars{3x/2} - sinpars{x/2}},ddpars{1 over x^{2}}
              end{align}




              Integrating by parts the last integral:
              begin{align}
              &int_{0}^{infty}{xcospars{x} - sinpars{x} over x^{3}},
              cospars{x over 2},dd x =
              \[5mm] & =
              -,{1 over 2}int_{0}^{infty}{1 - cospars{x/2} over x^{2}},dd x -
              {1 over 2}int_{0}^{infty}{1 - cospars{3x/2} over x^{2}},dd x
              \[5mm] & +
              {1 over 4}int_{x = 0}^{infty}{2 - 3cospars{3x/2}/2 - cospars{x/2}/2 over x^{2}},dd x
              \[1cm] & =
              -,{1 over 2}int_{0}^{infty}{1 - cospars{x/2} over x^{2}},dd x -
              {1 over 2}int_{0}^{infty}{1 - cospars{3x/2} over x^{2}},dd x
              \[5mm] & +
              {3 over 8}int_{0}^{infty}{1 - cospars{3x/2} over x^{2}},dd x +
              {1 over 8}int_{0}^{infty}{1 - cospars{x/2} over x^{2}},dd x
              \[1cm] & =
              -,{3 over 8}int_{0}^{infty}{1 - cospars{x/2} over x^{2}},dd x
              -,{1 over 8}int_{0}^{infty}{1 - cospars{3x/2} over x^{2}},dd x
              \[5mm] & =
              -,{3 over 16}int_{0}^{infty}{1 - cospars{x} over x^{2}},dd x
              -,{3 over 16}int_{0}^{infty}{1 - cospars{x} over x^{2}},dd x =
              -,{3 over 8}
              int_{0}^{infty}{1 - cospars{x} over x^{2}},dd x
              \[5mm] & =
              -,{3 over 4}int_{0}^{infty}{sin^{2}pars{x/2} over x^{2}},dd x =
              -,{3 over 8}
              underbrace{int_{0}^{infty}{sin^{2}pars{x} over x^{2}},dd x}
              _{ds{= {pi over 2}}} =
              bbox[#ffe,10px,border:1px dotted navy]{ds{-,{3 over 16},pi}}
              end{align}


              By integrating by parts:
              $ds{int_{0}^{infty}{sin^{2}pars{x} over x^{2}},dd x =
              int_{0}^{infty}{sinpars{x} over x},dd x = {1 over 2},pi}$.







              share|cite|improve this answer









              $endgroup$



              $newcommand{bbx}[1]{,bbox[8px,border:1px groove navy]{{#1}},}
              newcommand{braces}[1]{leftlbrace,{#1},rightrbrace}
              newcommand{bracks}[1]{leftlbrack,{#1},rightrbrack}
              newcommand{dd}{mathrm{d}}
              newcommand{ds}[1]{displaystyle{#1}}
              newcommand{expo}[1]{,mathrm{e}^{#1},}
              newcommand{ic}{mathrm{i}}
              newcommand{mc}[1]{mathcal{#1}}
              newcommand{mrm}[1]{mathrm{#1}}
              newcommand{pars}[1]{left(,{#1},right)}
              newcommand{partiald}[3][]{frac{partial^{#1} #2}{partial #3^{#1}}}
              newcommand{root}[2][]{,sqrt[#1]{,{#2},},}
              newcommand{totald}[3][]{frac{mathrm{d}^{#1} #2}{mathrm{d} #3^{#1}}}
              newcommand{verts}[1]{leftvert,{#1},rightvert}$
              begin{align}
              &int_{0}^{infty}{xcospars{x} - sinpars{x} over x^{3}},
              cospars{x over 2},dd x =
              int_{0}^{infty}{xbracks{1 - 2sin^{2}pars{x/2}} - sinpars{x} over x^{3}},cospars{x over 2},dd x
              \[5mm] & =
              {1 over 2}int_{0}^{infty}{2xcospars{x/2} - 2xsinpars{x}sinpars{x/2} -
              2sinpars{x}cospars{x/2} over x^{3}},,,dd x
              \[5mm] & =
              {1 over 2}int_{0}^{infty}{xcospars{x/2} + xcospars{3x/2} -
              sinpars{3x/2} - sinpars{x/2} over x^{3}},,,dd x
              \[1cm] & =
              -,{1 over 2}int_{0}^{infty}{1 - cospars{x/2} over x^{2}},dd x -
              {1 over 2}int_{0}^{infty}{1 - cospars{3x/2} over x^{2}},dd x
              \[5mm] & -
              {1 over 4}int_{x = 0}^{x to infty}bracks{2x - sinpars{3x/2} - sinpars{x/2}},ddpars{1 over x^{2}}
              end{align}




              Integrating by parts the last integral:
              begin{align}
              &int_{0}^{infty}{xcospars{x} - sinpars{x} over x^{3}},
              cospars{x over 2},dd x =
              \[5mm] & =
              -,{1 over 2}int_{0}^{infty}{1 - cospars{x/2} over x^{2}},dd x -
              {1 over 2}int_{0}^{infty}{1 - cospars{3x/2} over x^{2}},dd x
              \[5mm] & +
              {1 over 4}int_{x = 0}^{infty}{2 - 3cospars{3x/2}/2 - cospars{x/2}/2 over x^{2}},dd x
              \[1cm] & =
              -,{1 over 2}int_{0}^{infty}{1 - cospars{x/2} over x^{2}},dd x -
              {1 over 2}int_{0}^{infty}{1 - cospars{3x/2} over x^{2}},dd x
              \[5mm] & +
              {3 over 8}int_{0}^{infty}{1 - cospars{3x/2} over x^{2}},dd x +
              {1 over 8}int_{0}^{infty}{1 - cospars{x/2} over x^{2}},dd x
              \[1cm] & =
              -,{3 over 8}int_{0}^{infty}{1 - cospars{x/2} over x^{2}},dd x
              -,{1 over 8}int_{0}^{infty}{1 - cospars{3x/2} over x^{2}},dd x
              \[5mm] & =
              -,{3 over 16}int_{0}^{infty}{1 - cospars{x} over x^{2}},dd x
              -,{3 over 16}int_{0}^{infty}{1 - cospars{x} over x^{2}},dd x =
              -,{3 over 8}
              int_{0}^{infty}{1 - cospars{x} over x^{2}},dd x
              \[5mm] & =
              -,{3 over 4}int_{0}^{infty}{sin^{2}pars{x/2} over x^{2}},dd x =
              -,{3 over 8}
              underbrace{int_{0}^{infty}{sin^{2}pars{x} over x^{2}},dd x}
              _{ds{= {pi over 2}}} =
              bbox[#ffe,10px,border:1px dotted navy]{ds{-,{3 over 16},pi}}
              end{align}


              By integrating by parts:
              $ds{int_{0}^{infty}{sin^{2}pars{x} over x^{2}},dd x =
              int_{0}^{infty}{sinpars{x} over x},dd x = {1 over 2},pi}$.








              share|cite|improve this answer












              share|cite|improve this answer



              share|cite|improve this answer










              answered Oct 29 '16 at 6:42









              Felix MarinFelix Marin

              68.6k7109145




              68.6k7109145























                  7












                  $begingroup$

                  $$
                  begin{align}
                  &int_0^inftyfrac{xcos(x)-sin(x)}{x^3}cosleft(frac{x}{2}right),mathrm{d}xtag{1}\
                  &=int_0^inftyfrac{xleft(cosleft(frac32xright)+cosleft(frac12xright)right)-left(sinleft(frac32xright)+sinleft(frac12xright)right)}{2x^3},mathrm{d}xtag{2}\
                  &=-int_0^inftyfrac{xleft(cosleft(tfrac32xright)+cosleft(tfrac12xright)right)-left(sinleft(tfrac32xright)+sinleft(tfrac12xright)right)}{4},mathrm{d}x^{-2}tag{3}\
                  &=int_0^inftyfrac{left(frac12cosleft(frac12xright)-frac12cosleft(frac32xright)right)-xleft(frac32sinleft(frac32xright)+frac12sinleft(frac12xright)right)}{4x^2},mathrm{d}xtag{4}\
                  &=int_0^inftyleft(frac{1-cosleft(frac32xright)}{8x^2}-frac{1-cosleft(frac12xright)}{8x^2}-frac{3sinleft(frac32xright)}{8x}-frac{sinleft(frac12xright)}{8x}right)mathrm{d}x
                  tag{5}\
                  &=int_0^inftyleft(frac{3(1-cos(x))}{16x^2}-frac{1-cos(x)}{16x^2}-frac{3sin(x)}{8x}-frac{sin(x)}{8x}right)mathrm{d}x
                  tag{6}\
                  &=int_0^inftyleft(frac{3sin(x)}{16x}-frac{sin(x)}{16x}-frac{3sin(x)}{8x}-frac{sin(x)}{8x}right)mathrm{d}x
                  tag{7}\
                  &=-frac38int_0^inftyfrac{sin(x)}{x},mathrm{d}xtag{8}\
                  &=-frac{3pi}{16}tag{9}
                  end{align}
                  $$
                  Explanation:

                  $(2)$: trigonometric product formulas

                  $(3)$: prepare to integrate by parts

                  $(4)$: integrate by parts

                  $(5)$: separate integrals

                  $(6)$: substitute $xmapsto2x$ and $xmapstofrac23x$

                  $(7)$: integrate by parts

                  $(8)$: combine

                  $(9)$: $int_0^inftyfrac{sin(x)}{x},mathrm{d}x=fracpi2$






                  share|cite|improve this answer











                  $endgroup$













                  • $begingroup$
                    I see that this may be similar to Felix Marin's answer, but I think the path may be different enough, and possibly simpler, to warrant leaving it.
                    $endgroup$
                    – robjohn
                    Oct 29 '16 at 13:53


















                  7












                  $begingroup$

                  $$
                  begin{align}
                  &int_0^inftyfrac{xcos(x)-sin(x)}{x^3}cosleft(frac{x}{2}right),mathrm{d}xtag{1}\
                  &=int_0^inftyfrac{xleft(cosleft(frac32xright)+cosleft(frac12xright)right)-left(sinleft(frac32xright)+sinleft(frac12xright)right)}{2x^3},mathrm{d}xtag{2}\
                  &=-int_0^inftyfrac{xleft(cosleft(tfrac32xright)+cosleft(tfrac12xright)right)-left(sinleft(tfrac32xright)+sinleft(tfrac12xright)right)}{4},mathrm{d}x^{-2}tag{3}\
                  &=int_0^inftyfrac{left(frac12cosleft(frac12xright)-frac12cosleft(frac32xright)right)-xleft(frac32sinleft(frac32xright)+frac12sinleft(frac12xright)right)}{4x^2},mathrm{d}xtag{4}\
                  &=int_0^inftyleft(frac{1-cosleft(frac32xright)}{8x^2}-frac{1-cosleft(frac12xright)}{8x^2}-frac{3sinleft(frac32xright)}{8x}-frac{sinleft(frac12xright)}{8x}right)mathrm{d}x
                  tag{5}\
                  &=int_0^inftyleft(frac{3(1-cos(x))}{16x^2}-frac{1-cos(x)}{16x^2}-frac{3sin(x)}{8x}-frac{sin(x)}{8x}right)mathrm{d}x
                  tag{6}\
                  &=int_0^inftyleft(frac{3sin(x)}{16x}-frac{sin(x)}{16x}-frac{3sin(x)}{8x}-frac{sin(x)}{8x}right)mathrm{d}x
                  tag{7}\
                  &=-frac38int_0^inftyfrac{sin(x)}{x},mathrm{d}xtag{8}\
                  &=-frac{3pi}{16}tag{9}
                  end{align}
                  $$
                  Explanation:

                  $(2)$: trigonometric product formulas

                  $(3)$: prepare to integrate by parts

                  $(4)$: integrate by parts

                  $(5)$: separate integrals

                  $(6)$: substitute $xmapsto2x$ and $xmapstofrac23x$

                  $(7)$: integrate by parts

                  $(8)$: combine

                  $(9)$: $int_0^inftyfrac{sin(x)}{x},mathrm{d}x=fracpi2$






                  share|cite|improve this answer











                  $endgroup$













                  • $begingroup$
                    I see that this may be similar to Felix Marin's answer, but I think the path may be different enough, and possibly simpler, to warrant leaving it.
                    $endgroup$
                    – robjohn
                    Oct 29 '16 at 13:53
















                  7












                  7








                  7





                  $begingroup$

                  $$
                  begin{align}
                  &int_0^inftyfrac{xcos(x)-sin(x)}{x^3}cosleft(frac{x}{2}right),mathrm{d}xtag{1}\
                  &=int_0^inftyfrac{xleft(cosleft(frac32xright)+cosleft(frac12xright)right)-left(sinleft(frac32xright)+sinleft(frac12xright)right)}{2x^3},mathrm{d}xtag{2}\
                  &=-int_0^inftyfrac{xleft(cosleft(tfrac32xright)+cosleft(tfrac12xright)right)-left(sinleft(tfrac32xright)+sinleft(tfrac12xright)right)}{4},mathrm{d}x^{-2}tag{3}\
                  &=int_0^inftyfrac{left(frac12cosleft(frac12xright)-frac12cosleft(frac32xright)right)-xleft(frac32sinleft(frac32xright)+frac12sinleft(frac12xright)right)}{4x^2},mathrm{d}xtag{4}\
                  &=int_0^inftyleft(frac{1-cosleft(frac32xright)}{8x^2}-frac{1-cosleft(frac12xright)}{8x^2}-frac{3sinleft(frac32xright)}{8x}-frac{sinleft(frac12xright)}{8x}right)mathrm{d}x
                  tag{5}\
                  &=int_0^inftyleft(frac{3(1-cos(x))}{16x^2}-frac{1-cos(x)}{16x^2}-frac{3sin(x)}{8x}-frac{sin(x)}{8x}right)mathrm{d}x
                  tag{6}\
                  &=int_0^inftyleft(frac{3sin(x)}{16x}-frac{sin(x)}{16x}-frac{3sin(x)}{8x}-frac{sin(x)}{8x}right)mathrm{d}x
                  tag{7}\
                  &=-frac38int_0^inftyfrac{sin(x)}{x},mathrm{d}xtag{8}\
                  &=-frac{3pi}{16}tag{9}
                  end{align}
                  $$
                  Explanation:

                  $(2)$: trigonometric product formulas

                  $(3)$: prepare to integrate by parts

                  $(4)$: integrate by parts

                  $(5)$: separate integrals

                  $(6)$: substitute $xmapsto2x$ and $xmapstofrac23x$

                  $(7)$: integrate by parts

                  $(8)$: combine

                  $(9)$: $int_0^inftyfrac{sin(x)}{x},mathrm{d}x=fracpi2$






                  share|cite|improve this answer











                  $endgroup$



                  $$
                  begin{align}
                  &int_0^inftyfrac{xcos(x)-sin(x)}{x^3}cosleft(frac{x}{2}right),mathrm{d}xtag{1}\
                  &=int_0^inftyfrac{xleft(cosleft(frac32xright)+cosleft(frac12xright)right)-left(sinleft(frac32xright)+sinleft(frac12xright)right)}{2x^3},mathrm{d}xtag{2}\
                  &=-int_0^inftyfrac{xleft(cosleft(tfrac32xright)+cosleft(tfrac12xright)right)-left(sinleft(tfrac32xright)+sinleft(tfrac12xright)right)}{4},mathrm{d}x^{-2}tag{3}\
                  &=int_0^inftyfrac{left(frac12cosleft(frac12xright)-frac12cosleft(frac32xright)right)-xleft(frac32sinleft(frac32xright)+frac12sinleft(frac12xright)right)}{4x^2},mathrm{d}xtag{4}\
                  &=int_0^inftyleft(frac{1-cosleft(frac32xright)}{8x^2}-frac{1-cosleft(frac12xright)}{8x^2}-frac{3sinleft(frac32xright)}{8x}-frac{sinleft(frac12xright)}{8x}right)mathrm{d}x
                  tag{5}\
                  &=int_0^inftyleft(frac{3(1-cos(x))}{16x^2}-frac{1-cos(x)}{16x^2}-frac{3sin(x)}{8x}-frac{sin(x)}{8x}right)mathrm{d}x
                  tag{6}\
                  &=int_0^inftyleft(frac{3sin(x)}{16x}-frac{sin(x)}{16x}-frac{3sin(x)}{8x}-frac{sin(x)}{8x}right)mathrm{d}x
                  tag{7}\
                  &=-frac38int_0^inftyfrac{sin(x)}{x},mathrm{d}xtag{8}\
                  &=-frac{3pi}{16}tag{9}
                  end{align}
                  $$
                  Explanation:

                  $(2)$: trigonometric product formulas

                  $(3)$: prepare to integrate by parts

                  $(4)$: integrate by parts

                  $(5)$: separate integrals

                  $(6)$: substitute $xmapsto2x$ and $xmapstofrac23x$

                  $(7)$: integrate by parts

                  $(8)$: combine

                  $(9)$: $int_0^inftyfrac{sin(x)}{x},mathrm{d}x=fracpi2$







                  share|cite|improve this answer














                  share|cite|improve this answer



                  share|cite|improve this answer








                  edited Oct 29 '16 at 14:39

























                  answered Oct 29 '16 at 13:50









                  robjohnrobjohn

                  269k27311638




                  269k27311638












                  • $begingroup$
                    I see that this may be similar to Felix Marin's answer, but I think the path may be different enough, and possibly simpler, to warrant leaving it.
                    $endgroup$
                    – robjohn
                    Oct 29 '16 at 13:53




















                  • $begingroup$
                    I see that this may be similar to Felix Marin's answer, but I think the path may be different enough, and possibly simpler, to warrant leaving it.
                    $endgroup$
                    – robjohn
                    Oct 29 '16 at 13:53


















                  $begingroup$
                  I see that this may be similar to Felix Marin's answer, but I think the path may be different enough, and possibly simpler, to warrant leaving it.
                  $endgroup$
                  – robjohn
                  Oct 29 '16 at 13:53






                  $begingroup$
                  I see that this may be similar to Felix Marin's answer, but I think the path may be different enough, and possibly simpler, to warrant leaving it.
                  $endgroup$
                  – robjohn
                  Oct 29 '16 at 13:53













                  6












                  $begingroup$

                  We can also use contour integration.



                  $$ begin{align} &int_{0}^{infty} frac{x cos x - sin x}{x^{3}} , cos left(frac{x}{2} right) , dx \ &= frac{1}{2} int_{-infty}^{infty} frac{x left(frac{e^{ix}+e^{-ix}}{2} right) -frac{e^{ix}-e^{-ix}}{2i}}{x^{3}} left(frac{e^{ix/2}+e^{-ix/2}}{2} right) , dx \ &= frac{1}{2} lim_{epsilon to 0^{+}} int_{-infty}^{infty} frac{x left(frac{e^{ix}+e^{-ix}}{2} right) -frac{e^{ix}-e^{-ix}}{2i}}{(x- i epsilon)^{3}} left(frac{e^{ix/2}+e^{-ix/2}}{2} right) , dx \ &= frac{1}{8} lim_{epsilon to 0^{+}} int_{-infty}^{infty}frac{(x+i)(e^{3ix/2}+e^{ix/2})}{(x- iepsilon)^{3}} , dx + frac{1}{8} lim_{epsilon to 0^{+}} int_{-infty}^{infty} frac{(x-i)(e^{-ix/2}+e^{-3ix/2})}{(x- iepsilon)^{3}} , dx \ &=frac{1}{8} lim_{epsilon to 0^{+}} 2 pi i , text{Res} left[frac{(z+i)(e^{3iz/2}+e^{iz/2})}{(z- i epsilon)^{3}} , iepsilon right] + 0 tag{1} \ &= frac{1}{8} lim_{epsilon to 0^{+}} , 2 pi i , frac{1}{2!} lim_{z to i epsilon}frac{d^{2}}{dz^{2}} , (z+i)(e^{3iz/2}+e^{iz/2}) \ &= frac{1}{8} lim_{epsilon to 0^{+}} frac{pi}{4} , e^{-3 epsilon/2} left((epsilon-3) e^{epsilon} + 9 epsilon -3 right) \ &= - frac{3 pi}{16} end{align}$$





                  $(1)$ The second integral vanishes since the function $ displaystyle frac{(z-i)(e^{-iz/2}+e^{-3iz/2})}{(z- iepsilon)^{3}} $ is analytic in the lower half-plane where $left| e^{iaz} right| le 1$ if $a le 0$.






                  share|cite|improve this answer











                  $endgroup$


















                    6












                    $begingroup$

                    We can also use contour integration.



                    $$ begin{align} &int_{0}^{infty} frac{x cos x - sin x}{x^{3}} , cos left(frac{x}{2} right) , dx \ &= frac{1}{2} int_{-infty}^{infty} frac{x left(frac{e^{ix}+e^{-ix}}{2} right) -frac{e^{ix}-e^{-ix}}{2i}}{x^{3}} left(frac{e^{ix/2}+e^{-ix/2}}{2} right) , dx \ &= frac{1}{2} lim_{epsilon to 0^{+}} int_{-infty}^{infty} frac{x left(frac{e^{ix}+e^{-ix}}{2} right) -frac{e^{ix}-e^{-ix}}{2i}}{(x- i epsilon)^{3}} left(frac{e^{ix/2}+e^{-ix/2}}{2} right) , dx \ &= frac{1}{8} lim_{epsilon to 0^{+}} int_{-infty}^{infty}frac{(x+i)(e^{3ix/2}+e^{ix/2})}{(x- iepsilon)^{3}} , dx + frac{1}{8} lim_{epsilon to 0^{+}} int_{-infty}^{infty} frac{(x-i)(e^{-ix/2}+e^{-3ix/2})}{(x- iepsilon)^{3}} , dx \ &=frac{1}{8} lim_{epsilon to 0^{+}} 2 pi i , text{Res} left[frac{(z+i)(e^{3iz/2}+e^{iz/2})}{(z- i epsilon)^{3}} , iepsilon right] + 0 tag{1} \ &= frac{1}{8} lim_{epsilon to 0^{+}} , 2 pi i , frac{1}{2!} lim_{z to i epsilon}frac{d^{2}}{dz^{2}} , (z+i)(e^{3iz/2}+e^{iz/2}) \ &= frac{1}{8} lim_{epsilon to 0^{+}} frac{pi}{4} , e^{-3 epsilon/2} left((epsilon-3) e^{epsilon} + 9 epsilon -3 right) \ &= - frac{3 pi}{16} end{align}$$





                    $(1)$ The second integral vanishes since the function $ displaystyle frac{(z-i)(e^{-iz/2}+e^{-3iz/2})}{(z- iepsilon)^{3}} $ is analytic in the lower half-plane where $left| e^{iaz} right| le 1$ if $a le 0$.






                    share|cite|improve this answer











                    $endgroup$
















                      6












                      6








                      6





                      $begingroup$

                      We can also use contour integration.



                      $$ begin{align} &int_{0}^{infty} frac{x cos x - sin x}{x^{3}} , cos left(frac{x}{2} right) , dx \ &= frac{1}{2} int_{-infty}^{infty} frac{x left(frac{e^{ix}+e^{-ix}}{2} right) -frac{e^{ix}-e^{-ix}}{2i}}{x^{3}} left(frac{e^{ix/2}+e^{-ix/2}}{2} right) , dx \ &= frac{1}{2} lim_{epsilon to 0^{+}} int_{-infty}^{infty} frac{x left(frac{e^{ix}+e^{-ix}}{2} right) -frac{e^{ix}-e^{-ix}}{2i}}{(x- i epsilon)^{3}} left(frac{e^{ix/2}+e^{-ix/2}}{2} right) , dx \ &= frac{1}{8} lim_{epsilon to 0^{+}} int_{-infty}^{infty}frac{(x+i)(e^{3ix/2}+e^{ix/2})}{(x- iepsilon)^{3}} , dx + frac{1}{8} lim_{epsilon to 0^{+}} int_{-infty}^{infty} frac{(x-i)(e^{-ix/2}+e^{-3ix/2})}{(x- iepsilon)^{3}} , dx \ &=frac{1}{8} lim_{epsilon to 0^{+}} 2 pi i , text{Res} left[frac{(z+i)(e^{3iz/2}+e^{iz/2})}{(z- i epsilon)^{3}} , iepsilon right] + 0 tag{1} \ &= frac{1}{8} lim_{epsilon to 0^{+}} , 2 pi i , frac{1}{2!} lim_{z to i epsilon}frac{d^{2}}{dz^{2}} , (z+i)(e^{3iz/2}+e^{iz/2}) \ &= frac{1}{8} lim_{epsilon to 0^{+}} frac{pi}{4} , e^{-3 epsilon/2} left((epsilon-3) e^{epsilon} + 9 epsilon -3 right) \ &= - frac{3 pi}{16} end{align}$$





                      $(1)$ The second integral vanishes since the function $ displaystyle frac{(z-i)(e^{-iz/2}+e^{-3iz/2})}{(z- iepsilon)^{3}} $ is analytic in the lower half-plane where $left| e^{iaz} right| le 1$ if $a le 0$.






                      share|cite|improve this answer











                      $endgroup$



                      We can also use contour integration.



                      $$ begin{align} &int_{0}^{infty} frac{x cos x - sin x}{x^{3}} , cos left(frac{x}{2} right) , dx \ &= frac{1}{2} int_{-infty}^{infty} frac{x left(frac{e^{ix}+e^{-ix}}{2} right) -frac{e^{ix}-e^{-ix}}{2i}}{x^{3}} left(frac{e^{ix/2}+e^{-ix/2}}{2} right) , dx \ &= frac{1}{2} lim_{epsilon to 0^{+}} int_{-infty}^{infty} frac{x left(frac{e^{ix}+e^{-ix}}{2} right) -frac{e^{ix}-e^{-ix}}{2i}}{(x- i epsilon)^{3}} left(frac{e^{ix/2}+e^{-ix/2}}{2} right) , dx \ &= frac{1}{8} lim_{epsilon to 0^{+}} int_{-infty}^{infty}frac{(x+i)(e^{3ix/2}+e^{ix/2})}{(x- iepsilon)^{3}} , dx + frac{1}{8} lim_{epsilon to 0^{+}} int_{-infty}^{infty} frac{(x-i)(e^{-ix/2}+e^{-3ix/2})}{(x- iepsilon)^{3}} , dx \ &=frac{1}{8} lim_{epsilon to 0^{+}} 2 pi i , text{Res} left[frac{(z+i)(e^{3iz/2}+e^{iz/2})}{(z- i epsilon)^{3}} , iepsilon right] + 0 tag{1} \ &= frac{1}{8} lim_{epsilon to 0^{+}} , 2 pi i , frac{1}{2!} lim_{z to i epsilon}frac{d^{2}}{dz^{2}} , (z+i)(e^{3iz/2}+e^{iz/2}) \ &= frac{1}{8} lim_{epsilon to 0^{+}} frac{pi}{4} , e^{-3 epsilon/2} left((epsilon-3) e^{epsilon} + 9 epsilon -3 right) \ &= - frac{3 pi}{16} end{align}$$





                      $(1)$ The second integral vanishes since the function $ displaystyle frac{(z-i)(e^{-iz/2}+e^{-3iz/2})}{(z- iepsilon)^{3}} $ is analytic in the lower half-plane where $left| e^{iaz} right| le 1$ if $a le 0$.







                      share|cite|improve this answer














                      share|cite|improve this answer



                      share|cite|improve this answer








                      edited Oct 29 '16 at 13:15

























                      answered Oct 29 '16 at 8:05









                      Random VariableRandom Variable

                      25.6k172138




                      25.6k172138























                          4












                          $begingroup$

                          Inspired by Felix Marin's calculation using integration by parts.



                          Observe
                          begin{align}
                          int^infty_0 frac{xcos x-sin x}{x^3}cosfrac{x}{2} dx=& frac{1}{2}int^infty_{-infty} frac{xcos x-sin x}{x^3}e^{ix/2} dx\
                          =& frac{-1}{4} int^infty_{-infty} [xcos x-sin x] e^{ix/2} dleft(frac{1}{x^2} right).
                          end{align}
                          Using integration by parts, we have
                          begin{align}
                          frac{-1}{4} int^infty_{-infty} [xcos x-sin x] e^{ix/2} dleft(frac{1}{x^2} right)=& frac{1}{4} int^infty_{-infty}d([xcos x-sin x]e^{ix/2}) frac{1}{x^2}\
                          =& frac{-1}{4} int^infty_{-infty}frac{sin x}{x}e^{ix/2} dx + frac{i}{8} int^infty_{-infty} frac{xcos x-sin x}{x^2}e^{ix/2} dx.
                          end{align}
                          Now, observe
                          begin{align}
                          int^infty_{-infty}frac{sin x}{x}e^{ix/2} dx = mathcal{F}^{-1}left[operatorname{sincleft(frac{x}{pi}right)}right]left(frac{1}{2}right) = pi mathcal{F}^{-1}[operatorname{sinc}]left( frac{1}{2pi}right) = pi.
                          end{align}



                          Next, observe
                          begin{align}
                          int^infty_{-infty} frac{xcos x-sin x}{x^2} e^{ix/2} dx =& int^infty_{-infty} frac{d}{dx}left( frac{sin x}{x}right) e^{ix/2} dx\
                          =& -frac{i}{2}int^infty_{-infty}frac{sin x}{x} e^{ix/2} dx = -frac{ipi}{2}.
                          end{align}



                          Hence combining everything yields
                          begin{align}
                          int^infty_0 frac{xcos x-sin x}{x^3} cos frac{x}{2} dx = -frac{pi}{4} + frac{pi}{16} = -frac{3pi}{16}.
                          end{align}






                          share|cite|improve this answer











                          $endgroup$


















                            4












                            $begingroup$

                            Inspired by Felix Marin's calculation using integration by parts.



                            Observe
                            begin{align}
                            int^infty_0 frac{xcos x-sin x}{x^3}cosfrac{x}{2} dx=& frac{1}{2}int^infty_{-infty} frac{xcos x-sin x}{x^3}e^{ix/2} dx\
                            =& frac{-1}{4} int^infty_{-infty} [xcos x-sin x] e^{ix/2} dleft(frac{1}{x^2} right).
                            end{align}
                            Using integration by parts, we have
                            begin{align}
                            frac{-1}{4} int^infty_{-infty} [xcos x-sin x] e^{ix/2} dleft(frac{1}{x^2} right)=& frac{1}{4} int^infty_{-infty}d([xcos x-sin x]e^{ix/2}) frac{1}{x^2}\
                            =& frac{-1}{4} int^infty_{-infty}frac{sin x}{x}e^{ix/2} dx + frac{i}{8} int^infty_{-infty} frac{xcos x-sin x}{x^2}e^{ix/2} dx.
                            end{align}
                            Now, observe
                            begin{align}
                            int^infty_{-infty}frac{sin x}{x}e^{ix/2} dx = mathcal{F}^{-1}left[operatorname{sincleft(frac{x}{pi}right)}right]left(frac{1}{2}right) = pi mathcal{F}^{-1}[operatorname{sinc}]left( frac{1}{2pi}right) = pi.
                            end{align}



                            Next, observe
                            begin{align}
                            int^infty_{-infty} frac{xcos x-sin x}{x^2} e^{ix/2} dx =& int^infty_{-infty} frac{d}{dx}left( frac{sin x}{x}right) e^{ix/2} dx\
                            =& -frac{i}{2}int^infty_{-infty}frac{sin x}{x} e^{ix/2} dx = -frac{ipi}{2}.
                            end{align}



                            Hence combining everything yields
                            begin{align}
                            int^infty_0 frac{xcos x-sin x}{x^3} cos frac{x}{2} dx = -frac{pi}{4} + frac{pi}{16} = -frac{3pi}{16}.
                            end{align}






                            share|cite|improve this answer











                            $endgroup$
















                              4












                              4








                              4





                              $begingroup$

                              Inspired by Felix Marin's calculation using integration by parts.



                              Observe
                              begin{align}
                              int^infty_0 frac{xcos x-sin x}{x^3}cosfrac{x}{2} dx=& frac{1}{2}int^infty_{-infty} frac{xcos x-sin x}{x^3}e^{ix/2} dx\
                              =& frac{-1}{4} int^infty_{-infty} [xcos x-sin x] e^{ix/2} dleft(frac{1}{x^2} right).
                              end{align}
                              Using integration by parts, we have
                              begin{align}
                              frac{-1}{4} int^infty_{-infty} [xcos x-sin x] e^{ix/2} dleft(frac{1}{x^2} right)=& frac{1}{4} int^infty_{-infty}d([xcos x-sin x]e^{ix/2}) frac{1}{x^2}\
                              =& frac{-1}{4} int^infty_{-infty}frac{sin x}{x}e^{ix/2} dx + frac{i}{8} int^infty_{-infty} frac{xcos x-sin x}{x^2}e^{ix/2} dx.
                              end{align}
                              Now, observe
                              begin{align}
                              int^infty_{-infty}frac{sin x}{x}e^{ix/2} dx = mathcal{F}^{-1}left[operatorname{sincleft(frac{x}{pi}right)}right]left(frac{1}{2}right) = pi mathcal{F}^{-1}[operatorname{sinc}]left( frac{1}{2pi}right) = pi.
                              end{align}



                              Next, observe
                              begin{align}
                              int^infty_{-infty} frac{xcos x-sin x}{x^2} e^{ix/2} dx =& int^infty_{-infty} frac{d}{dx}left( frac{sin x}{x}right) e^{ix/2} dx\
                              =& -frac{i}{2}int^infty_{-infty}frac{sin x}{x} e^{ix/2} dx = -frac{ipi}{2}.
                              end{align}



                              Hence combining everything yields
                              begin{align}
                              int^infty_0 frac{xcos x-sin x}{x^3} cos frac{x}{2} dx = -frac{pi}{4} + frac{pi}{16} = -frac{3pi}{16}.
                              end{align}






                              share|cite|improve this answer











                              $endgroup$



                              Inspired by Felix Marin's calculation using integration by parts.



                              Observe
                              begin{align}
                              int^infty_0 frac{xcos x-sin x}{x^3}cosfrac{x}{2} dx=& frac{1}{2}int^infty_{-infty} frac{xcos x-sin x}{x^3}e^{ix/2} dx\
                              =& frac{-1}{4} int^infty_{-infty} [xcos x-sin x] e^{ix/2} dleft(frac{1}{x^2} right).
                              end{align}
                              Using integration by parts, we have
                              begin{align}
                              frac{-1}{4} int^infty_{-infty} [xcos x-sin x] e^{ix/2} dleft(frac{1}{x^2} right)=& frac{1}{4} int^infty_{-infty}d([xcos x-sin x]e^{ix/2}) frac{1}{x^2}\
                              =& frac{-1}{4} int^infty_{-infty}frac{sin x}{x}e^{ix/2} dx + frac{i}{8} int^infty_{-infty} frac{xcos x-sin x}{x^2}e^{ix/2} dx.
                              end{align}
                              Now, observe
                              begin{align}
                              int^infty_{-infty}frac{sin x}{x}e^{ix/2} dx = mathcal{F}^{-1}left[operatorname{sincleft(frac{x}{pi}right)}right]left(frac{1}{2}right) = pi mathcal{F}^{-1}[operatorname{sinc}]left( frac{1}{2pi}right) = pi.
                              end{align}



                              Next, observe
                              begin{align}
                              int^infty_{-infty} frac{xcos x-sin x}{x^2} e^{ix/2} dx =& int^infty_{-infty} frac{d}{dx}left( frac{sin x}{x}right) e^{ix/2} dx\
                              =& -frac{i}{2}int^infty_{-infty}frac{sin x}{x} e^{ix/2} dx = -frac{ipi}{2}.
                              end{align}



                              Hence combining everything yields
                              begin{align}
                              int^infty_0 frac{xcos x-sin x}{x^3} cos frac{x}{2} dx = -frac{pi}{4} + frac{pi}{16} = -frac{3pi}{16}.
                              end{align}







                              share|cite|improve this answer














                              share|cite|improve this answer



                              share|cite|improve this answer








                              edited Oct 29 '16 at 7:32

























                              answered Oct 29 '16 at 7:24









                              Jacky ChongJacky Chong

                              19.3k21129




                              19.3k21129























                                  3












                                  $begingroup$

                                  Integration by parts can be performed for the indefinite integral, using relations




                                  $$dfrac{xcos x-sin x}{x^2} = left(dfrac{sin x}{x}right)',quad
                                  sin^3 z =frac{3sin z-sin3z}4,quad cos^3z=frac{3cos
                                  z+cos3z}4.$$




                                  One can get
                                  begin{align}
                                  &int dfrac{xcos x-sin x}{x^3},cosdfrac x2, mathrm dx
                                  = intdfrac1{4sin frac x2},mathrm dleft(dfrac{sin x}{x}right)^2 \[4pt]
                                  &=dfrac1{4sin frac x2}left(dfrac{sin x}{x}right)^2
                                  +intleft(dfrac{sin x}{x}right)^2dfrac{cosfrac x2}{8sin^2 frac x2},mathrm dx
                                  =dfrac1{x^2}sinfrac x2,cos^2 frac x2
                                  +dfrac12intdfrac{cos^3frac x2}{x^2},mathrm dx\[4pt]
                                  &=dfrac1{x^2}sinfrac x2
                                  -dfrac1{4x^2}left(3sinfrac x2-sin frac {3x}2right)
                                  +dfrac18intdfrac{3cosfrac x2+cosfrac{3x}2}{x^2},mathrm dx\[4pt]
                                  &=dfrac1{4x^2}left(sinfrac x2+sin frac {3x}2right)
                                  -dfrac18intleft(3cosfrac x2+cosfrac{3x}2right),mathrm dfrac1x\[4pt]
                                  &=dfrac1{8x^2}left(2sinfrac x2+2sinfrac{3x}2-3xcosfrac {x}2-xcos frac {3x}2right)
                                  -dfrac3{16}intfrac1xleft(sinfrac x2+sinfrac{3x}2right),mathrm dx.
                                  end{align}

                                  Since
                                  $$limlimits_{xto0}dfrac{2sinfrac x2+2sinfrac{3x}2-3xcosfrac {x}2-xcosfrac {3x}2}{8x^2}
                                  = limlimits_{xto0}dfrac{2frac x2+2frac{3x}2-3x-x+O(x^3)}{8x^2}=0,$$




                                  $$intlimits_{0}^{infty} dfrac{sin x}x,mathrm dx =fracpi2,$$




                                  then
                                  $$intlimits_{0}^{infty} dfrac{xcos x-sin x}{x^3},cosdfrac x2, mathrm dx
                                  =-frac3{16}left(intlimits_{0}^{infty} dfrac{sinfrac x2}{frac x2},mathrm dfrac x2+intlimits_{0}^{infty} dfrac{sin frac{3x}2}{frac{3x}2},mathrm dfrac{3x}2right) = color{green}{mathbf{-frac{3pi}{16}}}.$$






                                  share|cite|improve this answer











                                  $endgroup$









                                  • 1




                                    $begingroup$
                                    elegant approach
                                    $endgroup$
                                    – G Cab
                                    Mar 15 at 22:53










                                  • $begingroup$
                                    @GCab Thanks. I tried to understand the essence of the problem.
                                    $endgroup$
                                    – Yuri Negometyanov
                                    Mar 15 at 23:06
















                                  3












                                  $begingroup$

                                  Integration by parts can be performed for the indefinite integral, using relations




                                  $$dfrac{xcos x-sin x}{x^2} = left(dfrac{sin x}{x}right)',quad
                                  sin^3 z =frac{3sin z-sin3z}4,quad cos^3z=frac{3cos
                                  z+cos3z}4.$$




                                  One can get
                                  begin{align}
                                  &int dfrac{xcos x-sin x}{x^3},cosdfrac x2, mathrm dx
                                  = intdfrac1{4sin frac x2},mathrm dleft(dfrac{sin x}{x}right)^2 \[4pt]
                                  &=dfrac1{4sin frac x2}left(dfrac{sin x}{x}right)^2
                                  +intleft(dfrac{sin x}{x}right)^2dfrac{cosfrac x2}{8sin^2 frac x2},mathrm dx
                                  =dfrac1{x^2}sinfrac x2,cos^2 frac x2
                                  +dfrac12intdfrac{cos^3frac x2}{x^2},mathrm dx\[4pt]
                                  &=dfrac1{x^2}sinfrac x2
                                  -dfrac1{4x^2}left(3sinfrac x2-sin frac {3x}2right)
                                  +dfrac18intdfrac{3cosfrac x2+cosfrac{3x}2}{x^2},mathrm dx\[4pt]
                                  &=dfrac1{4x^2}left(sinfrac x2+sin frac {3x}2right)
                                  -dfrac18intleft(3cosfrac x2+cosfrac{3x}2right),mathrm dfrac1x\[4pt]
                                  &=dfrac1{8x^2}left(2sinfrac x2+2sinfrac{3x}2-3xcosfrac {x}2-xcos frac {3x}2right)
                                  -dfrac3{16}intfrac1xleft(sinfrac x2+sinfrac{3x}2right),mathrm dx.
                                  end{align}

                                  Since
                                  $$limlimits_{xto0}dfrac{2sinfrac x2+2sinfrac{3x}2-3xcosfrac {x}2-xcosfrac {3x}2}{8x^2}
                                  = limlimits_{xto0}dfrac{2frac x2+2frac{3x}2-3x-x+O(x^3)}{8x^2}=0,$$




                                  $$intlimits_{0}^{infty} dfrac{sin x}x,mathrm dx =fracpi2,$$




                                  then
                                  $$intlimits_{0}^{infty} dfrac{xcos x-sin x}{x^3},cosdfrac x2, mathrm dx
                                  =-frac3{16}left(intlimits_{0}^{infty} dfrac{sinfrac x2}{frac x2},mathrm dfrac x2+intlimits_{0}^{infty} dfrac{sin frac{3x}2}{frac{3x}2},mathrm dfrac{3x}2right) = color{green}{mathbf{-frac{3pi}{16}}}.$$






                                  share|cite|improve this answer











                                  $endgroup$









                                  • 1




                                    $begingroup$
                                    elegant approach
                                    $endgroup$
                                    – G Cab
                                    Mar 15 at 22:53










                                  • $begingroup$
                                    @GCab Thanks. I tried to understand the essence of the problem.
                                    $endgroup$
                                    – Yuri Negometyanov
                                    Mar 15 at 23:06














                                  3












                                  3








                                  3





                                  $begingroup$

                                  Integration by parts can be performed for the indefinite integral, using relations




                                  $$dfrac{xcos x-sin x}{x^2} = left(dfrac{sin x}{x}right)',quad
                                  sin^3 z =frac{3sin z-sin3z}4,quad cos^3z=frac{3cos
                                  z+cos3z}4.$$




                                  One can get
                                  begin{align}
                                  &int dfrac{xcos x-sin x}{x^3},cosdfrac x2, mathrm dx
                                  = intdfrac1{4sin frac x2},mathrm dleft(dfrac{sin x}{x}right)^2 \[4pt]
                                  &=dfrac1{4sin frac x2}left(dfrac{sin x}{x}right)^2
                                  +intleft(dfrac{sin x}{x}right)^2dfrac{cosfrac x2}{8sin^2 frac x2},mathrm dx
                                  =dfrac1{x^2}sinfrac x2,cos^2 frac x2
                                  +dfrac12intdfrac{cos^3frac x2}{x^2},mathrm dx\[4pt]
                                  &=dfrac1{x^2}sinfrac x2
                                  -dfrac1{4x^2}left(3sinfrac x2-sin frac {3x}2right)
                                  +dfrac18intdfrac{3cosfrac x2+cosfrac{3x}2}{x^2},mathrm dx\[4pt]
                                  &=dfrac1{4x^2}left(sinfrac x2+sin frac {3x}2right)
                                  -dfrac18intleft(3cosfrac x2+cosfrac{3x}2right),mathrm dfrac1x\[4pt]
                                  &=dfrac1{8x^2}left(2sinfrac x2+2sinfrac{3x}2-3xcosfrac {x}2-xcos frac {3x}2right)
                                  -dfrac3{16}intfrac1xleft(sinfrac x2+sinfrac{3x}2right),mathrm dx.
                                  end{align}

                                  Since
                                  $$limlimits_{xto0}dfrac{2sinfrac x2+2sinfrac{3x}2-3xcosfrac {x}2-xcosfrac {3x}2}{8x^2}
                                  = limlimits_{xto0}dfrac{2frac x2+2frac{3x}2-3x-x+O(x^3)}{8x^2}=0,$$




                                  $$intlimits_{0}^{infty} dfrac{sin x}x,mathrm dx =fracpi2,$$




                                  then
                                  $$intlimits_{0}^{infty} dfrac{xcos x-sin x}{x^3},cosdfrac x2, mathrm dx
                                  =-frac3{16}left(intlimits_{0}^{infty} dfrac{sinfrac x2}{frac x2},mathrm dfrac x2+intlimits_{0}^{infty} dfrac{sin frac{3x}2}{frac{3x}2},mathrm dfrac{3x}2right) = color{green}{mathbf{-frac{3pi}{16}}}.$$






                                  share|cite|improve this answer











                                  $endgroup$



                                  Integration by parts can be performed for the indefinite integral, using relations




                                  $$dfrac{xcos x-sin x}{x^2} = left(dfrac{sin x}{x}right)',quad
                                  sin^3 z =frac{3sin z-sin3z}4,quad cos^3z=frac{3cos
                                  z+cos3z}4.$$




                                  One can get
                                  begin{align}
                                  &int dfrac{xcos x-sin x}{x^3},cosdfrac x2, mathrm dx
                                  = intdfrac1{4sin frac x2},mathrm dleft(dfrac{sin x}{x}right)^2 \[4pt]
                                  &=dfrac1{4sin frac x2}left(dfrac{sin x}{x}right)^2
                                  +intleft(dfrac{sin x}{x}right)^2dfrac{cosfrac x2}{8sin^2 frac x2},mathrm dx
                                  =dfrac1{x^2}sinfrac x2,cos^2 frac x2
                                  +dfrac12intdfrac{cos^3frac x2}{x^2},mathrm dx\[4pt]
                                  &=dfrac1{x^2}sinfrac x2
                                  -dfrac1{4x^2}left(3sinfrac x2-sin frac {3x}2right)
                                  +dfrac18intdfrac{3cosfrac x2+cosfrac{3x}2}{x^2},mathrm dx\[4pt]
                                  &=dfrac1{4x^2}left(sinfrac x2+sin frac {3x}2right)
                                  -dfrac18intleft(3cosfrac x2+cosfrac{3x}2right),mathrm dfrac1x\[4pt]
                                  &=dfrac1{8x^2}left(2sinfrac x2+2sinfrac{3x}2-3xcosfrac {x}2-xcos frac {3x}2right)
                                  -dfrac3{16}intfrac1xleft(sinfrac x2+sinfrac{3x}2right),mathrm dx.
                                  end{align}

                                  Since
                                  $$limlimits_{xto0}dfrac{2sinfrac x2+2sinfrac{3x}2-3xcosfrac {x}2-xcosfrac {3x}2}{8x^2}
                                  = limlimits_{xto0}dfrac{2frac x2+2frac{3x}2-3x-x+O(x^3)}{8x^2}=0,$$




                                  $$intlimits_{0}^{infty} dfrac{sin x}x,mathrm dx =fracpi2,$$




                                  then
                                  $$intlimits_{0}^{infty} dfrac{xcos x-sin x}{x^3},cosdfrac x2, mathrm dx
                                  =-frac3{16}left(intlimits_{0}^{infty} dfrac{sinfrac x2}{frac x2},mathrm dfrac x2+intlimits_{0}^{infty} dfrac{sin frac{3x}2}{frac{3x}2},mathrm dfrac{3x}2right) = color{green}{mathbf{-frac{3pi}{16}}}.$$







                                  share|cite|improve this answer














                                  share|cite|improve this answer



                                  share|cite|improve this answer








                                  edited Mar 15 at 22:47

























                                  answered Mar 15 at 22:16









                                  Yuri NegometyanovYuri Negometyanov

                                  12k1729




                                  12k1729








                                  • 1




                                    $begingroup$
                                    elegant approach
                                    $endgroup$
                                    – G Cab
                                    Mar 15 at 22:53










                                  • $begingroup$
                                    @GCab Thanks. I tried to understand the essence of the problem.
                                    $endgroup$
                                    – Yuri Negometyanov
                                    Mar 15 at 23:06














                                  • 1




                                    $begingroup$
                                    elegant approach
                                    $endgroup$
                                    – G Cab
                                    Mar 15 at 22:53










                                  • $begingroup$
                                    @GCab Thanks. I tried to understand the essence of the problem.
                                    $endgroup$
                                    – Yuri Negometyanov
                                    Mar 15 at 23:06








                                  1




                                  1




                                  $begingroup$
                                  elegant approach
                                  $endgroup$
                                  – G Cab
                                  Mar 15 at 22:53




                                  $begingroup$
                                  elegant approach
                                  $endgroup$
                                  – G Cab
                                  Mar 15 at 22:53












                                  $begingroup$
                                  @GCab Thanks. I tried to understand the essence of the problem.
                                  $endgroup$
                                  – Yuri Negometyanov
                                  Mar 15 at 23:06




                                  $begingroup$
                                  @GCab Thanks. I tried to understand the essence of the problem.
                                  $endgroup$
                                  – Yuri Negometyanov
                                  Mar 15 at 23:06











                                  1












                                  $begingroup$

                                  I use the Laplace transform method instead of Fourier because the first one is the main tool for an electrician (who I am) and the Laplace transform is very similar to the Fourier transform.



                                  By definition of the Laplace transform:



                                  $mathcal{L}f(x)=int_{0}^{infty}e^{-sx}f(x)dx=F(s)$



                                  $mathcal{L}g(x)=int_{0}^{infty}e^{-sx}g(x)dx=G(s)$



                                  Now we use the following equation from the Laplace transform theory:



                                  $$int_{0}^{infty}F(x)g(x)dx=int_{0}^{infty}G(x)f(x)dx$$



                                  and apply it to the required integral.



                                  We take



                                  $F(x)=frac{1}{x^3}$



                                  $g(x)=xcos xcos frac{x}{2}-sin xcos frac{x}{2}$



                                  and get after taking the Laplace and inverse Laplace transforms



                                  $f(x)=frac{x^2}{2}$



                                  $G(x)=frac{4x^2-3}{(4x^2+1)^2}-frac{4x^2+45}{(4x^2+9)^2}$



                                  Placing these results into the relationship above we arrive at the next expression for the required integral:



                                  $$I=frac{1}{2}int_{0}^{infty}x^2left [ frac{4x^2-3}{(4x^2+1)^2}-frac{4x^2+45}{(4x^2+9)^2} right ]dx$$



                                  $I=left [- frac{3}{16}arctan 2x- frac{3}{16}arctanfrac{2x}{3}+frac{9x}{4(4x^2+9)^2}+frac{x}{4(4x^2+1)^2}right ]_{0}^{infty}=$



                                  $=- frac{3}{16}frac{pi}{2}- frac{3}{16}frac{pi}{2}=- frac{3pi}{16}$






                                  share|cite|improve this answer









                                  $endgroup$


















                                    1












                                    $begingroup$

                                    I use the Laplace transform method instead of Fourier because the first one is the main tool for an electrician (who I am) and the Laplace transform is very similar to the Fourier transform.



                                    By definition of the Laplace transform:



                                    $mathcal{L}f(x)=int_{0}^{infty}e^{-sx}f(x)dx=F(s)$



                                    $mathcal{L}g(x)=int_{0}^{infty}e^{-sx}g(x)dx=G(s)$



                                    Now we use the following equation from the Laplace transform theory:



                                    $$int_{0}^{infty}F(x)g(x)dx=int_{0}^{infty}G(x)f(x)dx$$



                                    and apply it to the required integral.



                                    We take



                                    $F(x)=frac{1}{x^3}$



                                    $g(x)=xcos xcos frac{x}{2}-sin xcos frac{x}{2}$



                                    and get after taking the Laplace and inverse Laplace transforms



                                    $f(x)=frac{x^2}{2}$



                                    $G(x)=frac{4x^2-3}{(4x^2+1)^2}-frac{4x^2+45}{(4x^2+9)^2}$



                                    Placing these results into the relationship above we arrive at the next expression for the required integral:



                                    $$I=frac{1}{2}int_{0}^{infty}x^2left [ frac{4x^2-3}{(4x^2+1)^2}-frac{4x^2+45}{(4x^2+9)^2} right ]dx$$



                                    $I=left [- frac{3}{16}arctan 2x- frac{3}{16}arctanfrac{2x}{3}+frac{9x}{4(4x^2+9)^2}+frac{x}{4(4x^2+1)^2}right ]_{0}^{infty}=$



                                    $=- frac{3}{16}frac{pi}{2}- frac{3}{16}frac{pi}{2}=- frac{3pi}{16}$






                                    share|cite|improve this answer









                                    $endgroup$
















                                      1












                                      1








                                      1





                                      $begingroup$

                                      I use the Laplace transform method instead of Fourier because the first one is the main tool for an electrician (who I am) and the Laplace transform is very similar to the Fourier transform.



                                      By definition of the Laplace transform:



                                      $mathcal{L}f(x)=int_{0}^{infty}e^{-sx}f(x)dx=F(s)$



                                      $mathcal{L}g(x)=int_{0}^{infty}e^{-sx}g(x)dx=G(s)$



                                      Now we use the following equation from the Laplace transform theory:



                                      $$int_{0}^{infty}F(x)g(x)dx=int_{0}^{infty}G(x)f(x)dx$$



                                      and apply it to the required integral.



                                      We take



                                      $F(x)=frac{1}{x^3}$



                                      $g(x)=xcos xcos frac{x}{2}-sin xcos frac{x}{2}$



                                      and get after taking the Laplace and inverse Laplace transforms



                                      $f(x)=frac{x^2}{2}$



                                      $G(x)=frac{4x^2-3}{(4x^2+1)^2}-frac{4x^2+45}{(4x^2+9)^2}$



                                      Placing these results into the relationship above we arrive at the next expression for the required integral:



                                      $$I=frac{1}{2}int_{0}^{infty}x^2left [ frac{4x^2-3}{(4x^2+1)^2}-frac{4x^2+45}{(4x^2+9)^2} right ]dx$$



                                      $I=left [- frac{3}{16}arctan 2x- frac{3}{16}arctanfrac{2x}{3}+frac{9x}{4(4x^2+9)^2}+frac{x}{4(4x^2+1)^2}right ]_{0}^{infty}=$



                                      $=- frac{3}{16}frac{pi}{2}- frac{3}{16}frac{pi}{2}=- frac{3pi}{16}$






                                      share|cite|improve this answer









                                      $endgroup$



                                      I use the Laplace transform method instead of Fourier because the first one is the main tool for an electrician (who I am) and the Laplace transform is very similar to the Fourier transform.



                                      By definition of the Laplace transform:



                                      $mathcal{L}f(x)=int_{0}^{infty}e^{-sx}f(x)dx=F(s)$



                                      $mathcal{L}g(x)=int_{0}^{infty}e^{-sx}g(x)dx=G(s)$



                                      Now we use the following equation from the Laplace transform theory:



                                      $$int_{0}^{infty}F(x)g(x)dx=int_{0}^{infty}G(x)f(x)dx$$



                                      and apply it to the required integral.



                                      We take



                                      $F(x)=frac{1}{x^3}$



                                      $g(x)=xcos xcos frac{x}{2}-sin xcos frac{x}{2}$



                                      and get after taking the Laplace and inverse Laplace transforms



                                      $f(x)=frac{x^2}{2}$



                                      $G(x)=frac{4x^2-3}{(4x^2+1)^2}-frac{4x^2+45}{(4x^2+9)^2}$



                                      Placing these results into the relationship above we arrive at the next expression for the required integral:



                                      $$I=frac{1}{2}int_{0}^{infty}x^2left [ frac{4x^2-3}{(4x^2+1)^2}-frac{4x^2+45}{(4x^2+9)^2} right ]dx$$



                                      $I=left [- frac{3}{16}arctan 2x- frac{3}{16}arctanfrac{2x}{3}+frac{9x}{4(4x^2+9)^2}+frac{x}{4(4x^2+1)^2}right ]_{0}^{infty}=$



                                      $=- frac{3}{16}frac{pi}{2}- frac{3}{16}frac{pi}{2}=- frac{3pi}{16}$







                                      share|cite|improve this answer












                                      share|cite|improve this answer



                                      share|cite|improve this answer










                                      answered yesterday









                                      Martin GalesMartin Gales

                                      3,59411935




                                      3,59411935






























                                          draft saved

                                          draft discarded




















































                                          Thanks for contributing an answer to Mathematics Stack Exchange!


                                          • Please be sure to answer the question. Provide details and share your research!

                                          But avoid



                                          • Asking for help, clarification, or responding to other answers.

                                          • Making statements based on opinion; back them up with references or personal experience.


                                          Use MathJax to format equations. MathJax reference.


                                          To learn more, see our tips on writing great answers.




                                          draft saved


                                          draft discarded














                                          StackExchange.ready(
                                          function () {
                                          StackExchange.openid.initPostLogin('.new-post-login', 'https%3a%2f%2fmath.stackexchange.com%2fquestions%2f1989935%2fevaluating-the-improper-integral-int-0-infty-fracx-cos-x-sin-xx3-cos%23new-answer', 'question_page');
                                          }
                                          );

                                          Post as a guest















                                          Required, but never shown





















































                                          Required, but never shown














                                          Required, but never shown












                                          Required, but never shown







                                          Required, but never shown

































                                          Required, but never shown














                                          Required, but never shown












                                          Required, but never shown







                                          Required, but never shown







                                          Popular posts from this blog

                                          Magento 2 - Add success message with knockout Planned maintenance scheduled April 23, 2019 at 23:30 UTC (7:30pm US/Eastern) Announcing the arrival of Valued Associate #679: Cesar Manara Unicorn Meta Zoo #1: Why another podcast?Success / Error message on ajax request$.widget is not a function when loading a homepage after add custom jQuery on custom themeHow can bind jQuery to current document in Magento 2 When template load by ajaxRedirect page using plugin in Magento 2Magento 2 - Update quantity and totals of cart page without page reload?Magento 2: Quote data not loaded on knockout checkoutMagento 2 : I need to change add to cart success message after adding product into cart through pluginMagento 2.2.5 How to add additional products to cart from new checkout step?Magento 2 Add error/success message with knockoutCan't validate Post Code on checkout page

                                          Fil:Tokke komm.svg

                                          Where did Arya get these scars? Unicorn Meta Zoo #1: Why another podcast? Announcing the arrival of Valued Associate #679: Cesar Manara Favourite questions and answers from the 1st quarter of 2019Why did Arya refuse to end it?Has the pronunciation of Arya Stark's name changed?Has Arya forgiven people?Why did Arya Stark lose her vision?Why can Arya still use the faces?Has the Narrow Sea become narrower?Does Arya Stark know how to make poisons outside of the House of Black and White?Why did Nymeria leave Arya?Why did Arya not kill the Lannister soldiers she encountered in the Riverlands?What is the current canonical age of Sansa, Bran and Arya Stark?